Menu Close

Author: Tinku Tara

sin-101x-sin-90-x-dx-

Question Number 7385 by Tawakalitu. last updated on 25/Aug/16 $$\int{sin}\left(\mathrm{101}{x}\right)\:.\:{sin}^{\mathrm{90}} \left({x}\right)\:{dx} \\ $$ Commented by Yozzia last updated on 26/Aug/16 $${Let}\:{I}\left({n}\right)=\int{e}^{\mathrm{101}{ix}} {sin}^{{n}} {xdx}\:\:\:\left({n}\in\mathbb{Z}^{\geqslant} \:{i}=\sqrt{−\mathrm{1}}\right) \\…

v-x-f-x-f-x-mx-b-If-I-wish-to-rotate-v-counter-clockwise-by-degrees-how-does-one-do-so-where-v-is-rotated-about-the-origin-v-is-rotated-about-the-point-x-1-f-x-1-What-a

Question Number 7372 by FilupSmith last updated on 25/Aug/16 $$\boldsymbol{{v}}=\begin{bmatrix}{{x}}\\{{f}\left({x}\right)}\end{bmatrix},\:\:{f}\left({x}\right)={mx}+{b} \\ $$$$\mathrm{If}\:\mathrm{I}\:\mathrm{wish}\:\mathrm{to}\:\mathrm{rotate}\:\boldsymbol{{v}}\:\mathrm{counter}\:\mathrm{clockwise} \\ $$$$\mathrm{by}\:\theta\:\mathrm{degrees},\:\mathrm{how}\:\mathrm{does}\:\mathrm{one}\:\mathrm{do}\:\mathrm{so}\:\mathrm{where}: \\ $$$$\bullet\:\:\:\boldsymbol{{v}}\:\mathrm{is}\:\mathrm{rotated}\:\mathrm{about}\:\mathrm{the}\:\mathrm{origin} \\ $$$$\bullet\:\:\:\boldsymbol{{v}}\:\mathrm{is}\:\mathrm{rotated}\:\mathrm{about}\:\mathrm{the}\:\mathrm{point}\:\left({x}_{\mathrm{1}} ,\:{f}\left({x}_{\mathrm{1}} \right)\right) \\ $$$$\mathrm{What}\:\mathrm{are}\:\mathrm{the}\:\mathrm{new}\:\mathrm{vectors}? \\ $$ Answered…

f-x-0-and-lim-x-a-f-x-0-lim-x-a-g-x-then-lim-x-a-f-x-g-x-

Question Number 72905 by Tony Lin last updated on 04/Nov/19 $${f}\left({x}\right)\geqslant\mathrm{0},\:{and}\:\underset{{x}\rightarrow{a}} {\mathrm{lim}}{f}\left({x}\right)=\mathrm{0},\underset{{x}\rightarrow{a}} {\mathrm{lim}}{g}\left({x}\right)=\infty \\ $$$${then}\:\underset{{x}\rightarrow{a}} {\mathrm{lim}}{f}\left({x}\right)^{{g}\left({x}\right)} =? \\ $$ Commented by mathmax by abdo last…

n-0-1-2n-e-

Question Number 138437 by EnterUsername last updated on 13/Apr/21 $$\underset{\mathrm{n}=\mathrm{0}} {\overset{\infty} {\sum}}\frac{\mathrm{1}}{\left(\mathrm{2n}\right)!!}=\sqrt{\mathrm{e}} \\ $$ Answered by Ar Brandon last updated on 13/Apr/21 $$\:\:\:\:\:\left(\mathrm{2n}\right)!!=\mathrm{2n}\left(\mathrm{2n}−\mathrm{2}\right)\left(\mathrm{2n}−\mathrm{4}\right)…\mathrm{2} \\ $$$$\:\:\:\:\:\:\:\:\:\:\:\:\:\:\:\:\:\:=\mathrm{2}^{\mathrm{n}}…